The speed of light is 186,282 miles per second write the place name and the value of each eight in this number

Answers

Answer 1

Place name of eights are ten thousands place & tens place & 80000, 80 are the values respectively.

The expanded form of a number is splitting of numbers based on place value. The place are ones, tens, hundreds, thousands, ten thousand etc

that is, the number represented by the sum of each digit multiplied by its place value is called the expanded form of a number.

Example : Expanded form of 4567 is 4000+500+60+7. where 4 is in thousands place, 5 is in hundreds place, 6 is in tens place and 7 is in ones place.

Given that, The speed of light is 186,282 miles per second.

The number is 186282.

Expanding the number, we get,

186282 = 100000+80000+6000+200+80+2

Here in the number 186282, eight present in the ten thousands place and in the tens place.

Since one eight is in ten thousands place, so the value is 80000

And the another eight is in the tens place, so the value is 80.

Learn more about places of number here :

https://brainly.com/question/1713079

#SPJ10


Related Questions

Suppose that shifting into third gear decreased the engine speed (rpm) while increasing the tractor speed (mph). What would be the effect of this change?
a). both area rate and flow rate would increase
b). area rate would increase and flow rate decrease
c). arear rate would decrease and flow rate increase
d). both area rate and flow rate would increase

Answers

What would be the effect of this change is: a). both area rate and flow rate would increase.

Shifting of gear

Assuming a driver shift into third gear in which there was reduction in the engine speed while on the other hand the tractor speed increase.

The effect of the change will result in the increase in area rate as well as the  flow rate.

Therefore the correct option  is A.

Learn more about Shifting of gear here:https://brainly.com/question/18192699

#SPJ1

what is the difference between 15

Answers

Answer:

its an odd number

Step-by-step explanation:

i need help plss helpp

Answers

The equation of the line in the graph given is: y = -3x - 1.

How to Write the Equation of a Line?

Find the slope of the line, which is: m = rise/run.

Rise of the line = 3 units

Run = 1

Slope (m) = -3/1 = -3

Y-intercept (b) = -1 (at this point, x equals zero)

Substitute m = -3 and b = -1 into y = mx + b

y = -3x + (-1)

y = -3x - 1

Learn more about equation of a line on:

https://brainly.com/question/13763238

#SPJ1

In theparallelogram, the value of x is ?

Answers

Answer:

x = 6 cm

Step-by-step explanation:

in the parallelogram, if we consider the side of length 12

  as the base then the corresponding height is 5.

In this case the area of our parallelogram = 5 × 12 = 60 cm²

………………………………………………

in the parallelogram, if we consider the side of length 10

  as the base then the corresponding height is x.

In this case the area of our parallelogram = 10 × x = 10x cm²

………………………………………………

In order to determine x ,we equate the two different results

that we got for the same area .

In other words ,we have to solve the equation 10x = 60.

10x = 60

⇔ x = 60/10 = 6

Answer:

6 cm

Step-by-step explanation:

the 2 triangles are similar, we can solve with an equation

12 : 10 = x : 5

x = 12 * 5 : 10

x = 60 : 10

x = 6 cm

Can someone help me with these 6 problems I need them within 9 mins. I’m currently in class it says to (Find the slope of each line)

Answers

See below for the calculations of the slopes of the graphs

How to determine the slope?

The slope of a line is calculated using:

m= (y2 - y1)/(x2 - x1)

Using the above formula, we have:

Line 1: (-2, 0) and (3, 2)

Slope, m= (2 - 0)/(3 + 2)

m= 2/5

Hence, the slope is 2/5

Line 2: (2, -3) and (-2, 4)

Slope, m= (4 + 3)/(-2 - 2)

m= -7/4

Hence, the slope is -7/4

Line 3: (-4, -2) and (1, 3)

Slope, m= (3 + 2)/(1 + 4)

m= 1

Hence, the slope is 1

Line 4: (0, 0) and (-3, -4)

Slope, m= (-4 - 0)/(-3 - 0)

m= 4/3

Hence, the slope is 4/3

Line 5: (0, 0) and (-1, -4)

Slope, m= (-4 - 0)/(-1 - 0)

m= 4

Hence, the slope is 4

Line 6: (-2, 1) and (3, 2)

Slope, m= (2 - 1)/(3 + 2)

m= 1/5

Hence, the slope is 1/5

Line 7: (-2, -1) and (2, -4)

Slope, m= (-4 + 1)/(2 + 2)

m= -3/4

Hence, the slope is -3/4

Line 8: (-2, -3) and (-1, -4)

Slope, m= (-4 + 3)/(-1 + 2)

m= -1

Hence, the slope is -1

Read more about slopes at:

https://brainly.com/question/3493733

#SPJ1

Select the graph that correctly displays this system of equations and point of intersection.

Answers

The solution of the equations will be the point of intersection of the two lines

Equation of a line

A line is the shortest distance between two points. The equation of a line in standard form is given as Ax + By = C

According to the question, we are to pot the following lines.

4y +3x = 0

4y - x = 16

The solution of the equations will be the point of intersection of the two lines as attached.

Learn more on equation of a line here:https://brainly.com/question/13763238

#SPJ1

Solve for x. Triangle ABC is similar to Triangle FED

Answers

Answer: x=9.

Step-by-step explanation:

Triangle ABC is similar to triangle FED     ⇒

[tex]\frac{77}{21} =\frac{11x+11}{30} \\\\\frac{7*11}{7*3}=\frac{11*(x+1)}{30} \\\\\frac{11}{3}*30 =11*(x+1)\\\\\frac{11*3*10}{3} =11*(x+1)\\\\11*10=11*(x+1)\\10=x+1\\9=x.[/tex]

Good luck an' have a nice day!

5 Quick algebra 1 questions for 50 points!


Only answer if you know all 5, Tysm! :)

Answers

answers

6. y = -1/2x + 8

7. y = 15

8. y = -3/4x + 1

9. y = 6x + 3

10. y = 1/3x - 4

steps

equation of the line that is perpendicular means the number next to the x should be a negative reciprocal

number next to the x is the slope

6. y=2x+5

y = mx + b

m = 2

b = -5

negative reciprocal would be

-1/2x

m = -1/2

(2, 7) means x = 2 y = 7

substitute

7 = -1/2(2) + b

7 = -1 + b

b = 8

answer is y = -1/2x + 8

7.

y = 5; (11, 15)

b = 5

m would be 0 because there is no slope

15 = 0*11 + b

b = 15

y = 15

8.

line goes through

(0,-2) and (3,2)

get slope

y2-y1/x2-x1

2-(-2) / 3 - 0

4/3

y = 4/3x

point slope form

y - y1 = m (x-x1)

y - -2 =  4/3 (x - 0)

y+2 =  4/3x

y = 4/3x - 2

negative reciprocal slope would be for perpendicular

-3/4x

(12, 10)

y = mx + b

10 = 3/4(12) + b

10 = 9 + b

1 =  b

b = 1

y = -3/4x + 1

9.

y = -1/6x + 1; (− 2, — 9)

negative reciprocal slope would be for perpendicular

+6x

(− 2, — 9)

y = mx + b

get b

-9 = 6(-2) + b

-9 = -12 + b

b = 3

y = 6x + 3

10.

6x+2y=14; (12, 0)

2y= -6x+14

y= -3x+7

negative reciprocal slope would be for perpendicular

1/3x

y = mx + b

get b

0 = 1/3(12) + b

0 = 4 + b

b = -4

y = 1/3x - 4

Consider the equality xy k. Write the following inverse proportion: y is inversely proportional to x. When y = 12, x=5.​

Answers

Answer:

[tex]y=\dfrac {60} {x}[/tex]   or   [tex]xy=60[/tex]   (depending on your teacher's format preference)

Step-by-step explanation:

Proportionality background

Proportionality is sometimes called "variation".   (ex. " 'y' varies inversely as 'x' ")

There are two main types of proportionality/variation:

DirectInverse.

Every proportionality, regardless of whether it is direct or inverse, will have a constant of proportionality (I'm going to call it "k").

Below are several different examples of both types of proportionality, and how they might be stated in words:

[tex]y=kx[/tex]      y is directly proportional to x[tex]y=kx^2[/tex]     y is directly proportional to x squared[tex]y=kx^3[/tex]     y is directly proportional to x cubed[tex]y=k\sqrt{x}}[/tex]   y is directly proportional to the square root of x[tex]y=\dfrac {k} {x}[/tex]   y is inversely proportional to x[tex]y=\dfrac {k} {x^2}[/tex]   y is inversely proportional to x squared

From these examples, we see that two things:

things that are directly proportional -- the thing is multiplied to the constant of proportionality "k"things that are inversely proportional -- the thing is divided from the constant of proportionality "k".

Looking at our question

In our question, y is inversely proportional to x, so the equation we're looking at is the following [tex]y=\dfrac {k} {x}[/tex].

It isn't yet clear what the constant of proportionality "k" is for this situation, but we are given enough information to solve for it:  "When y=12, x=5."

We can substitute this known relationship pair, and find the "k" that relates this pair of numbers:

Solving for k, and finding the general equation

General Inverse variation equation...

[tex]y=\dfrac {k} {x}[/tex]

Substituting known values...

[tex](12)=\dfrac {k} {(5)}[/tex]

Multiplying both sides by 5...

[tex](12)*5= \left ( \dfrac {k} {5} \right ) *5[/tex]

Simplifying/arithmetic...

[tex]60=k[/tex]

So, for our situation, k=60.  So the inverse proportionality relationship equation for this situation is [tex]y=\dfrac {60} {x}[/tex].

The way your question is phrased, they may prefer the form: [tex]xy=60[/tex]

HELP ME PLEASEEE 20 POINTSSS!!!!
Fighter pilots the maximum height, h, of a fighter pilot is 77 inches. Write this as an inequality.

h ≤ 70

h ≥ 77

h ≤ 77

h ≥ 70

Answers

Answer:

h ≤ 70 I'm pretty sure I could be wrong

The correct inequality for the maximum height, h, of a fighter pilot is h ≤ 77.

Given that the height of the fighter pilot (h) must be less than or equal to 77 inches.

The symbol "≤" represents "less than or equal to," and 77 inches is the maximum allowable height for a fighter pilot.

The inequality h ≥ 77 would be incorrect since it states that the height of the fighter pilot should be greater than or equal to 77 inches, which would not be a maximum height requirement.

Similarly, the inequality h ≤ 70 would also be incorrect as it would imply a height limit of 70 inches, which is lower than the actual maximum height allowed.

Lastly, h ≥ 70 would not be suitable as it would permit heights greater than or equal to 70 inches, potentially exceeding the specified maximum height of 77 inches.

In conclusion, the correct inequality to represent the maximum height of a fighter pilot is h ≤ 77, ensuring that their height does not exceed 77 inches.

To learn more about inequality;

brainly.com/question/28823603

#SPJ3

Everyday Math
574. The population of the United States on July 4,2010 was almost 310,000,000. Write the number in scientific notation.

Answers

Answer:

If the population of the United States on July 4,2010 was almost 310000000 then, the number in scientific notation is [tex]$3.1 \times 10^{8}$[/tex].

Step-by-step explanation:

- Given

Population of the United States on July 4,2010 was almost 310000000 .

- To write the number in scientific notation.

- Firstly, move the decimal point so that the first factor is greater than or equal to 1 but less than 10 .

- Then, count the number of decimal places, n, that the decimal point was moved.

- Then, write the number as a product with a power of 10.

- Finally, check the scientific notation.

Step 1 of 2

Moving the decimal point-

Move the decimal after 3 . so that 3.10000000 is between 1 and 10 .

Now, we know the decimal point was moved 8 places to the left.

Step 2 of 2

Writing the number as a product with a power of 10 -

Since, 310000000 is greater than 1 so the power of 10 will have the exponent 8 .

Therefore, the number becomes-

[tex]$$31000000=3.1 \times 10^{8}$$[/tex]

Now, checking the scientific notation-

[tex]$$\begin{gathered}3.1 \times 10^{8}=3.1 \times 100000000 \\=310000000\end{gathered}$$[/tex]

4 Quick algebra 1 question for 50 points!

Only answer if you know the answer, tysm!

Define the domain and range of each function: (in the pictures)

Answers

A)

[tex]f(x )= - 2x[/tex]

Domain: ( - ∞ , ∞ ) or ] -∞ , ∞ [Range: ( - ∞ , ∞ ) or ] -∞ , ∞ [

B)

[tex]g(x) = \frac{1}{2} x {}^{2} + 5[/tex]

Domain: ( - ∞ , ∞ ) or ] -∞ , ∞ [Range: [ 5 , ∞ [ or [ 5 , ∞ )

C)Domain: ( - ∞ , ∞ ) or ] -∞ , ∞ [Range: ( - ∞ , ∞ ) or ] -∞ , ∞ [

D)Domain: ( - ∞ , ∞ ) or ] -∞ , ∞ [Range: ( - ∞ , - 3 ] or ] - ∞ , - 3 ]

#1

y=-2x

Domain

(-oo,oo)

Range

(-oo,oo)

#2

y=0.5x²+5

Domain

(-oo,oo)

Range

[5,oo)

#3

Its graphed

Domain=Range=(-oo,oo)

#4

Domain

(-oo,oo)

Range

[-3,-oo) or (-oo,-3]

Rectangular prism B is the image of rectangular prism A after dilation by a scale factor of \frac{1}{2} 2 1 ​ . If the volume of rectangular prism B is 8 m^3 3 , find the volume of rectangular prism A, the preimage.

Answers

The quantities which are squares or multiple of linear things twice grow by square of scale factor. The volume of rectangular prism A = 64 m³

How does dilation affect length, area, and volume of an object?

Suppose a figure (pre image) is dilated (dilated image) by scale factor of k.

So, if a side of the figure is of length L units, and that of its similar figure is of M units, then:

L = k × M

where 'k' will be called as scale factor.

The linear things grow linearly like length, height etc.

The quantities which are squares or multiple of linear things twice grow by square of scale factor. Thus, we need to multiply or divide by k² to get each other corresponding quantity from their similar figures' quantities.

So, area of first figure = k² × area of second figure

Similarly, Volume of first figure = k³ × volume of second figure.

It is because we will need to multiply 3 linear quantities to get volume, which results in k getting multiplied 3 times, thus, cubed.


Given that the Rectangular prism B is the image of rectangular prism A after dilation by a scale factor of 1/2.

Scale factor =  (Length of rectangular prism B)/(Length of rectangular prism A)

1/2 = (Length of rectangular prism B)/(Length of rectangular prism A)

Now, the volume of the rectangular prism can be written as,

(Scale factor)³ = (Volume of rectangular prism B)/(Volume of rectangular prism A)

(1/2)³ = 8 m³ /(Volume of rectangular prism A)

The volume of rectangular prism A = 64 m³

Hence, The volume of rectangular prism A = 64 m³

Learn more about Dilation:

https://brainly.com/question/27212783

#SPJ1

d
an and David are marking exam papers. Each set takes Dan 35 minutes and David 1 hour. Express the times Dan and David take as a ratio.
Give your answer in its simplest form.

Answers

Answer:5

Step-by-step explanation:

Having some trouble here need some help not really good at this

Answers

Answer:

none of these are correct

Step-by-step explanation:

Please solve each of the following by factoring!

Answers

Answer:

see explanation

Step-by-step explanation:

(a)

x² - 36 = 0 ← is a difference of squares and factors as

(x - 6)(x + 6) = 0

equate each factor to zero and solve for x

x + 6 = 0 ⇒ x = - 6

x - 6 = 0 ⇒ x = 6

(b)

x² - 5x + 4 = 0

consider the product of the factors of the constant term (+ 4) which sum to give the coefficient of the x- term (- 5)

the factors are - 1 and - 4 , since

- 1 × - 4 = + 4 and - 1 - 4 = - 5 , then

(x - 1)(x - 4) = 0 ← in factored form

equate each factor to zero and solve for x

x - 1 = 0 ⇒ x = 1

x - 4 = 0 ⇒ x = 4

(c)

x² - 2x = 3 ( subtract 3 from both sides )

x² - 2x - 3 = 0 ← in standard form

consider the product of the factors of the constant term (- 3) which sum to give the coefficient of the x- term (- 2)

the factors are + 1 and - 3 , since

1 × - 3 = - 3 and 1 - 3 = - 2 , then

(x + 1)(x - 3) = 0 ← in factored form

equate each factor to zero and solve for x

x + 1 = 0 ⇒ x = - 1

x - 3 = 0 ⇒ x = 3

(d)

6x² - 11x - 10 = 0

consider the product of the factors of the coefficient of the x² term and the constant term which sum to give the coefficient of the x- term.

product = 6 × - 10 = - 60 and sum = - 11

the factors are + 4 and - 15

use these factors to split the x- term

6x² + 4x - 15x - 10 = 0 ( factor the first/second and third/fourth terms )

2x(3x + 2) - 5(3x + 2) = 0 ← factor out (3x + 2) from each term

(3x + 2)(2x - 5) = 0

equate each factor to zero and solve for x

3x + 2 = 0 ⇒ 3x = - 2 ⇒ x = - [tex]\frac{2}{3}[/tex]

2x - 5 = 0 ⇒ 2x = 5 ⇒ x = [tex]\frac{5}{2}[/tex]

PLEASE HURRY!!!!!!!!
Mr. Jackson has decided to open a new health club in his town. To decide whether he will have enough members to be successful, he hires a researcher to conduct a survey. What is the population from which the researcher should choose a sample?

A. People who exercise regularly.
B. Members of other health clubs in the town.
C. Trainers at other health clubs.
D. His friends and family.

Answers

Answer:

I would say b since this would tell him how many people are wiling to join a health club

Hope This Helps!!!

A pianist plans to play 4 pieces at a recital from her repertoire of 20 pieces, and is carefully considering which song to play first, second, etc. to create a good flow. How many different recital programs are possible

Answers

There are 1,16,280 different recital programs possible.

Given: A pianist plans to play 4 pieces at a recital from her repertoire of 20 pieces and is carefully considering which song to play first, second, etc.

It means the repetition of pieces is not allowed.

When repetition is not allowed, then the number of ways to arrange n things where r things taken together is given by:-

[tex]^{n} P_{r} =\frac{n!}{(n-r)!}[/tex]

Now, the number of different recital programs are:-

[tex]^{20} P_{4} =\frac{20!}{(20-4)!}=\frac{20*19*18*17*16!}{16!}=116280[/tex]

Hence, there are 1,16,280 different recital programs possible.

Learn more about probability here https://brainly.com/question/251701

#SPJ4

Which equation has infinite solutions?
3(x – 1) = x + 2(x + 1) + 1
x – 4(x + 1) = –3(x + 1) + 1
2x + 3 = 2 x plus 3 equals StartFraction one-half EndFraction left-parenthesis 4 x plus 2 right-parenthesis plus 2.(4x + 2) + 2
StartFraction one-third EndFraction left-parenthesis 6 x minus 3 right-parenthesis equals 3 left-parenthesis x plus 1 right-parenthesis minus x minus 2.(6x – 3) = 3(x + 1) – x – 2

Answers

The equation that has an infinite number of solutions is [tex]2x + 3 = \frac{1}{2}(4x + 2) + 2[/tex]

How to determine the equation?

An equation that has an infinite number of solutions would be in the form

a = a

This means that both sides of the equation would be the same

Start by simplifying the options

3(x – 1) = x + 2(x + 1) + 1

3x - 3 = x + 3x + 2 + 1

3x - 3 = 4x + 3

Evaluate

x = 6 ----- one solution

x – 4(x + 1) = –3(x + 1) + 1

x - 4x - 4 = -3x - 3 + 1

-3x - 4 = -3x - 2

-4 = -2 ---- no solution

[tex]2x + 3 = \frac{1}{2}(4x + 2) + 2[/tex]

2x + 3 = 2x + 1 + 2

2x + 3 = 2x + 3

Subtract 2x

3 = 3 ---- infinite solution

Hence, the equation that has an infinite number of solutions is [tex]2x + 3 = \frac{1}{2}(4x + 2) + 2[/tex]

Read more about equations at:

https://brainly.com/question/15349799

#SPJ1

Complete question

Which equation has infinite solutions?

3(x – 1) = x + 2(x + 1) + 1

x – 4(x + 1) = –3(x + 1) + 1

[tex]2x + 3 = \frac{1}{2}(4x + 2) + 2[/tex]

[tex]\frac 13(6x - 3) = 3(x + 1) - x - 2[/tex]

Question 4 of 5
Use the drawing tools to form the correct answer on the number line.
Graph the solution set to this inequality.
-4(1 + 3) ≤ -21
Drawing Tools
Select
Point
Open Point
Ray
>
Click on a tool to begin drawing.
A+++
-10
-8
-6
-4
-2
0
Delete
2
4
Undo
6
20.
8
Reset
10
?
os

Answers

The solution set of -4(x + 3) ≤ -21 is x ≥ 2.25

How to graph the solution set?

The inequality is given as:

-4(x + 3) ≤ -21

Divide both sides by -4

x + 3 ≥ 5.25

Subtract 3 from both sides

x ≥ 2.25

This means that the solution set of -4(x + 3) ≤ -21 is x ≥ 2.25

See attachment for the number line

Read more about inequality at:

https://brainly.com/question/25275758

#SPJ1

Can I please get a thorough way of how solve this question I don't understand what to do.

Answers

Answer:

HI: 17.9

GH: 22.1

Step-by-step explanation:

1) There are three trigonometric ratios for right-angled triangles.

Sin = opposite/hypotenuse

Cos = adjacent/hypotenuse

Tan = opposite/adjacent.

2) Opposite to the 90° lies the hypotenuse. Opposite to the given angle lies the opposite.

Beside the given angle lies the adjacent.

3) The value of G given to us is 54°. The length of GI is 13. We are asked to find the lengths of HI and GH. We cannot find GH (we can, but I prefer the current method) without the length of HI. Let's find HI.

Tan(54) = x/13

Tan(54) × 13 = x

x = 17.89

4) We can use the Pythagorean theorem to find the length of GH. His theorem is c² = a² + b².

c² = a² + b²

c² = 13² + 17.89²

c² = 489.0521

c = √489.0521

c = 22.11

5) Round off the answers to the nearest tenth.

HI = 17.9

GH = 22.1

Wonder waves aquarium just opened a new tropics wing, with a large main tank and a small observation tank. there are 75 fish in the large tank, 25 of which are angelfish. there are 25 fish in the small tank, 5 of which are angelfish. do the tanks have the same ratio of angelfish to total fish?

Answers

Answer:

No, the ratio is not the same.

Step-by-step explanation:

75 total (large tank) / 25 total (small tank) = 3

25 angelfish (large tank) / 5 angelfish (small tank) = 5

The ratio of angelfish to total fish is not the same. When we scale up from the small tank to large tank in terms of total fish, we have a factor of 3. However, when we scale up from the small tank to large tank in terms of angelfish, we have a factor of 5. For the ratio to be the same, these factors would need to be identical.

1011 base 2 -1001 base 2​

Answers

Answer:0

Step-by-step explanation:

Everyday Math
426. Heating Oil A 275 gallon oil tank costs $400 to fill. How much would it cost to fill a 180 gallon oil tank?

Answers

Answer:

261.81818181

Step-by-step explanation:

400÷275=x

x×180

Answer:

If 275 gallon of oil tank costs $400 to fill.

Then 180 gallon of oil tank costs $261.8 to fill.

Step-by-step explanation:

Given that a 275 gallon of oil tank costs $400 to fill.We need to find how much it would cost to fill a 180 gallon oil tank.Use the unitary method to find the answer.

Step 1 of 2

If 275  gallon of oil tank costs $400  to fill.

Then the cost to fill 180  gallon of oil tank can be found by forming a proportion.

Let the required cost be x.

[tex]$$\begin{aligned}\frac{\text { cost }}{\text { Gallons of Gas }} &=\frac{\text { cost }}{\text { Gallons of Gas }} \\\frac{\$ 400}{275} &=\frac{x}{180}\end{aligned}[/tex]

Multiplying both sides by 180 we get,

[tex]$$\begin{aligned}\frac{\$ 400}{275} \times 180 &=\frac{x}{180} \times 180 \\x &=\frac{\$ 400}{55} \times 36 \\x &=\$ 261.8\end{aligned}$$[/tex]

Step 2 of 2

To check if the answer is reasonable, we substitute it back in the formed proportion

[tex]$\frac{\$ 400}{275}=\frac{x}{180}$$[/tex]

We found x=$261.8 hence,

[tex]$\frac{\$ 400}{275}=\frac{\$ 261.8}{180}$$$\$ 1.45=\$ 1.45$$Here, $L H S=R H S$[/tex]

Hence, our answer is correct.

Identifying Transformations On a coordinate plane, triangle A has points (3, 3), (6, 3), (6, 9). Triangle A prime has points (negative 2, negative 1), (negative 2, negative 2), (negative 3, negative 2). Determine all the transformations needed to transform the pre-image, A, to the image, A'. Select all that apply. dilation by a scale factor of 1/2 dilation by a scale factor of 1/3 reflection over the x-axis translation to the left 4 units translation down 3 units

Answers

Triangle A was dilated by a scale factor of 1/3 and reflected over the x axis to form triangle A'

What is a transformation?

Transformation is the movement of a point from its initial location to a new location. Types of transformation are translation, rotation, reflection and dilation.

Dilation is the increase or decrease in the size of a figure.

Triangle A was dilated by a scale factor of 1/3 and reflected over the x axis to form triangle A'

Find out more on transformation at: brainly.com/question/4289712

#SPJ1

Answer:

b d e

Step-by-step explanation:

an airplane flies in the path shown by the vector line UV on the graph below. what is the magnitude and direction of line UV? (1 unit represents 1 mile)

Answers

The magnitude and direction of line UV is 19.7° South of West for 8.1 miles

How to find the magnitude of the vector line UV?

The magnitude of the vector line UV is given by the length of the line UV,

d = √[(x₂ - x₁)² + (y₂ - y₁)²] where

(x₁, y₁) = (2, 3) and (x₂, y₂) = (-2, -4)

Substituting the values of the variables intot he equation, we have

d = √[(x₂ - x₁)² + (y₂ - y₁)²]

d = √[(-2 - 2)² + (-4 - 3)²]

d = √[(-4)² + (-7)²]

d = √[4² + 7²]

d = √[16 + 49]

d = √65

d = 8.06 miles

d ≅ 8.1 miles

So, the magnitude of vector line UV is 8.1 miles

How to find the direction of the vector line UV

The direction of the vector line UV is given by Ф = tan⁻¹[(y₂ - y₁)/(x₂ - x₁)] where

(x₁, y₁) = (2, 3) and (x₂, y₂) = (-2, -4)

Substituting the values of the variables into the equation, we have

Ф = tan⁻¹[(y₂ - y₁)/(x₂ - x₁)]

Ф = tan⁻¹[(-4 - 3)/(-2 - 2)]

Ф = tan⁻¹[(-7)/(-4)]

Ф = tan⁻¹[7/4]

Ф = tan⁻¹[1.75]

Ф = 60.26°

Ф ≅ 60.3°

Its bearing from the north-south line is α = 90° - Ф

= 90° - 60.3°

= 19.7°

So, its direction is 19.7° South of West

So, the magnitude and direction of line UV is 19.7° South of West for 8.1 miles

Learn more about vectors here:

https://brainly.com/question/26700114

#SPJ1

Use trigonometry to work out the height, v,
of the equilateral triangle below.
Give your answer in millimetres to 1 d.p.
9.5 mm
v

Answers

Answer: 8.2 mm

Step-by-step explanation:

[tex]\sin 60^{\circ}=\frac{v}{9.5}\\\\v=9.5 \sin 60^{\circ} \approx \boxed{8.2 \text{ mm}}[/tex]

Find the value of z.

Answers

Answer:

z = 6[tex]\sqrt{2}[/tex]

Step-by-step explanation:

using the altitude- on- hypotenuse theorem

(leg of large Δ )² = (part of hypotenuse below it ) × (whole hypotenuse)

z² = 8 × (8 + 1) = 8 × 9 = 72 ( take square root of both sides )

z = [tex]\sqrt{72}[/tex] = [tex]\sqrt{36(2)}[/tex] = [tex]\sqrt{36}[/tex] × [tex]\sqrt{2}[/tex] = 6[tex]\sqrt{2}[/tex]

Answer:

[tex]z = 6 \sqrt{2} [/tex]

Step-by-step explanation:

The solution is in the image

1/2x+1/3y=9 and 3/5x-3/4y=-3
use elimination method​

Answers

(You could do this shorter, but by the time i realized that it was too late.)


First, we need one of the variables (x or y) to have the same coefficient.
Lets use the variable y, since it uses smaller numbers.

Multiply the first equation by the denominator of y, which is 3. Then multiply the second equation by the denominator of y in the second equation, which is 4.

You get:
1st equation: [tex]\frac{3}{2} x+ y = 27[/tex]
2nd equation: [tex]\frac{12}{5}x - 3y=-12[/tex]
Now we need to multiply the first equation by 3, so we can have "3y" in both of the equations.
1st equation: 9/2x + 3y = 81
2nd equation: [tex]\frac{12}{5}x - 3y=-12[/tex]
Now we can subtract both of the equations.
12/5x + 9/2x = 69 We need to make the common denominator for x.
24/10x + 45/10x = 69
69/10x=69
x=10
Now we substitute x into the first equation (or the second, doesn't matter).
1/2 * 10 + 1/3y=9
5 + 1/3y = 9
1/3 y = 4
y = 12

Answer:

(10, 12 )

Step-by-step explanation:

let's begin by clearing the fractions from both equations

[tex]\frac{1}{2}[/tex] x + [tex]\frac{1}{3}[/tex] y = 9 ( multiply through by 6 ( the LCM of 2 and 3 to clear ) )

3x + 2y = 54 → (1)

[tex]\frac{3}{5}[/tex] x - [tex]\frac{3}{4}[/tex] y = - 3 ( multiply through by 20 ( the LCM of 5 and 4 to clear ) )

12x - 15y = - 60 → (2)

multiplying (1) by - 4 and adding to (2) will eliminate x )

- 12x - 8y = - 216 → (3)

add (2) and (3) term by term to eliminate x

0 - 23y = - 276

- 23y = - 276 ( divide both sides by - 23 )

y = 12

substitute y - 12 into either of the 2 equations and solve for x

substituting into (1)

3x + 2(12) = 54

3x + 24 = 54 ( subtract 24 from both sides )

3x = 30 ( divide both sides by 3 )

x = 10

solution is (10, 12 )

Which of the following functions shows the quadratic parent function, F(x)= x2, vertically compressed?
A. G(x) = (2.5x)²
B. G(x) = x²
C. G(x) = (14x)²
D. G(x) = 5x²

Answers

If the function is vertically compressed, the resulting function will be g(x) = 3/4x^2

Vertical compression of a function

Quadratic functions are functions that has a leading degree of 2. Given the parent function f(x) = ax^2

where

2 is the exponent/degree

If the function is vertically compressed, the value of a must be less than 1, hence the possible expression when vertically compressed is g(x) = 3/4x^2

Learn more on vertical stretch here:  https://brainly.com/question/14325265

#SPJ1

Other Questions
Terms:Length of lease = 48 months MSRP of the car = $32,100Purchase value of the car after lease = $26,200Down payment = $4400Monthly payment = $450$435 security deposit$530 acquisition fee. HELP ASAP!!!When the polynomial 2x^3 + mx^2 + nx 2 is divided by x 3, the remainder is 4. Whendivided by x 2, the remainder is 2. What are the values of m and n? On october 1, sam reported the formal marketing/sales event he has scheduled for 9 a.m. on november 19. luckily, before he had flyers printed, he remembered he has an 8 a.m. dentist appointment november 19. just to make sure he can get to his event on time, he advertises the event to start at 9:30 a.m. what event reporting infraction(s) might sam incur A tub of water is emptied at a rate of 3 gallons per minute. The equation y 12 = 3(x 1) models the amount of water remaining, where x is time (in minutes) and y is the amount of water left (in gallons). Analyze the work shown below to determine the initial amount of water. 1. Solve for the y-variable. y 12 = 3(x 1) y 12 = 3x + 3 y = 3x +15 2. Write the equation using function notation. f(x) = 3x +15 The tub started with gallons of water. 14. the function y=-16t^2+ 50t +4 describes the height (in feet) of a discus t seconds after it is released. describe the domain and range of the function. find the maximum height of the discus. A sample of size 400 has mean =6.0. Can it be regarded as a sample from a large population with mean 6.25 inches and standard deviation 2.25 inches ? using =0.05 One reason that multiple approaches are often taken to answer a testable question is that scientistshave different educational backgrounds and experiences.want to determine the single best way to find an answer.often look for the quickest way to answer the question.get tired of using the same approach every single time. TIME SENSITIVE When doing upper body exercises which of the following should be performed first?A. Military pressB. Lat pull downC. Overhead pressD. Incline bench . A wheel of diameter 0.2 m starts from rest and accelerated with constant angular acceleration to an angular velocity of 900 rev min in 5 s. (a) Find the position at the end of 1 s of a point originally at the top of the wheel. (b) Compute and show in a diagram the magnitude and the direction of the acceleration at the end of 1 s. Can someone help please? Triangle not drawn to scaleGiven: m&A= 62,m&C=28,c=24cm. Then a_?__cm to the nearest hundredth. (PLEASE HELP FAST!!!) Which expression gives the area of triangle QRS? How does the author introduce his thesis?A. with a direct statement followed by examplesB. with examples followed by an introductory questionC. with an introductory question followed by evidenceD. with evidence followed by a direct statement Psychotherapy for psychological disorders is generally more effective than no treatment at all. true or false What goes around and comes around Why are samples obtained from participant pools maintained by psychology departments considered more representative of college students in general than volunteer samples from the college population How were senators chosen during the time of the Roman Republic?A.They were elected by the people.B.They were chosen by the consuls.C.They were selected by the gods.D.They were appointed by the censors. after Vietnam was divided at a peace conference in Geneva: A. the United States supported the anticommunist leader Ngo Dinh Diem. B. the United States supported the quest for Vietnamese independence. C. the United States remained neutral. D. Ngo Dinh Diem's rule was widely accepted. Tina baked a batch of 40 cupcakes. daisy baked 4/5 as many cupcakes as tina. miko baked 1/2 as many cupcakes as daisy. how many cupcakes did the 3 girls bake altogether? Jessica and francis are best friends. Francis exercises every day, but jessica is only active once in a while. Based on the information you know, who is more likely to go to college?. Beginning with the rise of the Gracchi brothers in the late 2nd century B.C.E., discuss the factors that led to the collapse of the Roman Republic at the end of the 1st century B.C.E.